You are on page 1of 22

MODULE V

EQUIVALENT RESISTANCE AND


KIRCHHOFF’S LAWS

Lesson 1 Resistors in Series and


in Parallel

Lesson 2 Resistors in Series-


Parallel and Parallel-
Series

Lesson 3 Kirchhoff’s Laws

Module V
2

MODULE V

EQUIVALENT RESISTANCE AND KIRCHHOFF’S LAWS

 INTRODUCTION

This module presents equivalent resistance and Kirchhoff’s laws. It will


discuss the following topics on resistors connected in series and in parallel,
resistors combination and Kirchhoff’s voltage and current law.

OBJECTIVES

After studying the module, you should be able to:

1. Identify resistors connected in series and in parallel.


2. Determine equivalent resistance, voltage and current to circuits
connected in series and parallel.
3. Solve and analyze worded problem and determine current and
voltage in each resistor.
4. Evaluate complex circuits connected in Series-Parallel / Parallel-
Series and construct a single circuit.
5. Define and identify node, junction, loop and branch in a circuit.
6. Solve and analyze given circuits and apply Kirchhoff’s current law
and voltage law.
7. Determine the branch current and terminal voltage of the
batteries.

 DIRECTIONS/ MODULE ORGANIZER

There are three lessons in the module. Read each lesson thoughtfully
then answer the learning activities and a summative test at the end of the
module to find out how much you have benefited from it. Work on these
activities carefully. Date of submission and how will be submitted will be
posted in the google classroom or group chat.
All activities required can be printed or copied in a coupon bond.
Solutions/answers will be handwritten.
On the paper, write first your family name then your first name on the
blank provided fill up and then affix your signature over printed name on the
right bottom part (all pages).
In case you encounter difficulty, contact or message your
instructor/professor through messenger or cellphone no. 09186205795.

Good luck and happy reading!!

Module V
3

Lesson 1

 Resistors in Series and


in Parallel

RESISTORS IN SERIES: When current can follow only one path as it flows
through two or more resistors connected in line, the resistors are in series. In
other words, when one ad only one terminal of a resistor is connected directly
to one and only one terminal of another resistor, the two are in series and
the same current passes through both. A node is a point where three or more
current-carrying wires or branches meet. There are no nodes between circuit
elements (such as capacitors, resistors, and batteries) that are connected in
series. A typical case is seen in Figure (a) below. For several resistors in series,
their equivalent resistance, 𝑅𝑒𝑞 is given by

𝑅𝑒𝑞 = 𝑅1 + 𝑅2 + 𝑅3 + ⋯

Where 𝑅1 + 𝑅2 + 𝑅3 …, are the resistances of the several resistors.

In a series combination, the current through each resistance is the


same as that through all the others. The potential drop (p.d.) across the
combination is equal to the sum of the individual potential drops. The
equivalent resistance in series is always greater than the largest of the
individual resistances.

𝐼𝑒𝑞 = 𝐼1 = 𝐼2 = 𝐼3 = ⋯

𝑉𝑒𝑞 = 𝑉1 + 𝑉2 + 𝑉3 + ⋯

RESISTORS IN PARALLEL: Several resistors are connected in parallel


between two nodes if one end of each is connected to one node and the other
end of each is connected to the other node. A typical case is shown in above
Figure b, where points a and b are nodes. Their equivalent resistance, 𝑅𝑒𝑞 is
given by

1 1 1 1
= + + +⋯
𝑅𝑒𝑞 𝑅1 𝑅2 𝑅3

Module V
4

The equivalent resistance in parallel is always less than the smallest


of the individual resistances. Connecting additional resistances in parallel
decreases Req for the combination.

The potential drop V across each resistor in a parallel combination is


the same as the potential drop across each of the others. The current through
the nth resistor is In = V/Rn and the total current entering the combination is
equal to the sum of the individual branch currents

𝐼𝑒𝑞 = 𝐼1 + 𝐼2 + 𝐼3 + ⋯

𝑉𝑒𝑞 = 𝑉1 = 𝑉2 = 𝑉3 = ⋯

Example Problems:

1. Compute the equivalent resistance of 4 Ω and 8 Ω (a) in series, (b) in


parallel.

(𝑎) 𝑅𝑒𝑞 = 𝑅1 + 𝑅2 = 4 Ω + 8 Ω = 12 Ω

1 1 1 1 1
(𝑏) = + or 𝑅𝑒𝑞 = = = 2.67 Ω
𝑅𝑒𝑞 𝑅1 𝑅2 1 1 1 1
+ +
𝑅1 𝑅2 4Ω 8Ω

𝑅1 𝑥 𝑅2 4Ω𝑥8Ω
𝑜𝑟 (𝑏) 𝑅𝑒𝑞 = = = 2.67 Ω
𝑅1 + 𝑅2 4 Ω + 8 Ω

2. Compute the equivalent resistance of 3 Ω, 6 Ω and 9 Ω (a) in series,


(b) in parallel.

(𝑎) 𝑅𝑒𝑞 = 𝑅1 + 𝑅2 + 𝑅3 = 3 Ω + 6 Ω + 9 Ω = 18 Ω

1 1 1 1
(𝑏) = + + 𝑜𝑟
𝑅𝑒𝑞 𝑅1 𝑅2 𝑅3

1 1
(𝑏) 𝑅𝑒𝑞 = = = 1.64 Ω
1 1 1 1 1 1
+ + + +
𝑅1 𝑅2 𝑅3 3Ω 6Ω 9Ω

3. A 120 𝑉 house circuit has the following light bulbs turned on: 40 𝑊,
60 𝑊 and 75 𝑊. Find the equivalent resistance of the lights.

House circuits are so connected that each device is connected in


parallel with the others. From 𝑃 = 𝑉𝐼 = 𝑉 2 ⁄𝑅 , we have for the first
bulb

𝑉2 (120 𝑉)2
𝑅1 = = = 360 Ω
𝑃1 40. 0 𝑊

Module V
5

Similarly, 𝑅2 = 240 Ω and 𝑅3 = 192 Ω. Because they are in parallel

1 1 1 1
= + + 𝑜𝑟 𝑅𝑒𝑞 = 82.29 Ω
𝑅𝑒𝑞 360 Ω 240 Ω 192 Ω

As a check, we note that the total power drawn from the line is 40 𝑊
+ 60 𝑊 + 75 𝑊 = 175 𝑊.

Then using 𝑃 = 𝑉 2 ⁄𝑅

𝑉2 (120 𝑉)2
𝑅𝑒𝑞 = = = 82.29 Ω
𝑡𝑜𝑡𝑎𝑙 𝑝𝑜𝑤𝑒𝑟 175 𝑊

4. What resistance must be placed in parallel with 12 Ω to obtain a


combined resistance of 4 Ω?

1 1 1
= +
𝑅𝑒𝑞 𝑅1 𝑅2

1 1 1
= +
4 Ω 12 Ω 𝑅2

𝑅2 = 6 Ω

5. Several 40 Ω resistors are to be connected so that 15 𝐴 flows from a


120 𝑉 source. How can this be done? How many resistors are
connected?

The equivalent resistance must be such that 15 𝐴 flows from


a 120 𝑉. Thus

𝑉 120 𝑉
𝑅𝑒𝑞 = = =8Ω
𝐼 15 𝐴
The resistors must be in parallel, since the combined resistance
is to be smaller than any of them. If the required number of 40 Ω
resistors is 𝑛, then we have

1 1
=𝑛 𝑜𝑟 𝑛=5
8Ω 40 Ω

6. As shown in below figure (a) a battery (internal resistance 1 Ω) is


connected in series with two resistors. Compute (a) the current in the
circuit, (b) the p.d. across each resistor, (c) the terminal p.d. of the
battery.

Module V
6

The circuit is redrawn in fig. (b) so as to show the battery resistance.


(Note that internal resistance 𝑟 is included in solving the equivalent
resistance.) We have

𝑅𝑒𝑞 = 5 Ω + 12 Ω + 1 Ω = 18 Ω

Hence the circuit is equivalent to the one shown in fig. (c). Applying
𝑉 = 𝐼𝑅 to it, we have

𝑉 18 𝑉
(𝑎) 𝐼= = = 1.0 𝐴
𝑅𝑒𝑞 18 Ω

(b) Since 𝐼 = 1.0 𝐴, we can find the p.d. from point b to c as

𝑉𝑏𝑐 = 𝐼𝑅𝑏𝑐 = (1.0 𝐴)(12 Ω) = 12 𝑉

and that from c to d as

𝑉𝑐𝑑 = 𝐼𝑅𝑐𝑑 = (1.0 𝐴)(5 Ω) = 5 𝑉

Notice that 𝐼 is the same at all points.

(c) The terminal p.d. of the battery is the p.d. from a to e.

Therefore,

𝑇𝑒𝑟𝑚𝑖𝑛𝑎𝑙 𝑝. 𝑑. = 𝑉𝑏𝑐 + 𝑉𝑐𝑑 = 12 𝑉 + 5 𝑉 = 17 𝑉

Or, we could start at e and keep track of the voltage changes as we


go through the battery from e to a. Taking voltage drops as negative,
we have

𝑇𝑒𝑟𝑚𝑖𝑛𝑎𝑙 𝑝. 𝑑. = −(𝐼𝑟) + Ԑ = −(1.0 𝐴)(1 Ω) + 18 𝑉 = 17 𝑉

7. For the situation shown in the figure, find the current 𝐼 in each
resistor and the current drawn from the battery.

Module V
7

Notice that the p.d. from a to b is 40 V. Therefore, the p.d. across


each resistor is 40 V. Then,

40 𝑉 40 𝑉 40 𝑉
𝐼2 = = 20 𝐴 𝐼5 = = 8.0 𝐴 𝐼8 = = 5.0 𝐴
2.0 Ω 5.0 Ω 8.0 Ω
Because 𝐼 splits into three currents, the battery current is

𝐼 = 𝐼2 + 𝐼5 + 𝐼8 = 20 𝐴 + 8.0 𝐴 + 5.0 𝐴 = 33 𝐴

Module V
8

 LEARNING ACTIVITY
Name&Section:_______________________________Date:______Score:____
Subject: ENSC 102 LA#11 Instructor: Engr. Mary Jean C. Armiendo

Direction: At the space below, show your complete solution to gain full
credits. Wrong solution means incorrect answer. Box your final answer. Use
this page as your answer sheet. You may use additional sheets if necessary.

1. Compute the equivalent resistance of (a) 3, 4, 7, 10 and 12 Ω in parallel,


(b) three 33 Ω heating elements in parallel, (c) twenty 100 Ω lamps in
parallel.
2. What resistance must be placed in parallel with 20 Ω to make the
combined resistance 15 Ω?
3. How many 160 Ω resistors (in parallel) are required to carry 5 𝐴 on a
100 𝑉 line?
4. Three resistors of 8, 12, and 24 Ω are in parallel and a current of 20 𝐴 is
drawn by the combination. Determine (a) the potential difference
across the combination and (b) the current in 8 Ω (c) the current in
12 Ω and (d) the current in 24 Ω.
5. A series circuit consists of resistors 40, 60 𝑎𝑛𝑑 20 Ω are connected to a
120 V source. Calculate the (a) total resistance (b) battery current (c)
current in each resistor (d) voltage in each resistor.

Module V
9

Lesson 2

 Resistors in Series-Parallel
and Parallel-Series

RESISTORS COMBINATION: several resistors are connected in series-


parallel and parallel-series.

Series-Parallel Connected Resistors

Series-parallel circuit- a combinational circuit which when


simplified will result into a series circuit.

Parallel- Series Connected Resistors

Parallel- series circuit- a combinational circuit which when


simplified will result into a parallel circuit.

Example Problems:

1. Figure (a) is in series- parallel resistors, determine the equivalent


resistance and the current I through the battery.

The 3.0 Ω and 7.0 Ω resistors are in parallel; their joint resistance 𝑅1 is
found from

Module V
10

1 1 1 10
= + = 𝑜𝑟 𝑅1 = 2.1 Ω
𝑅1 3.0 Ω 7.0 Ω 21 Ω

Then the equivalent resistance of the entire circuit is

𝑅𝑒𝑞 = 2.1 Ω + 5 Ω + 0.4 Ω = 7.5 Ω

Note: the internal resistance r = 0.4 Ω is included in solving for the


equivalent resistance of the circuit.

And the battery current is

Ԑ 30 𝑉
𝐼= = = 4.0 𝐴
𝑅𝑒𝑞 7.5 Ω

2. Figure (b) in in series- parallel resistors, determine the equivalent


resistance and the current I through the battery.

The 7.0 Ω, 1.0 Ω and 10.0 Ω resistors are in parallel; their joint
resistance is 18.0 Ω. Then 18.0 Ω is in parallel with 6.0 Ω; their
combined resistance 𝑅1 is given by

1 1 1
= + 𝑜𝑟 𝑅1 = 4.5 Ω
𝑅1 18.0 Ω 6.0 Ω

Hence, the equivalent resistance of the entire circuit is

𝑅𝑒𝑞 = 4.5 Ω + 2.0 Ω + 8.0 Ω + 0.3 Ω = 14.8 Ω

And the battery current is

Ԑ 20 𝑉
𝐼= = = 1.4 𝐴
𝑅𝑒𝑞 14.8 Ω

3. In the figure, the battery has an internal resistance of 0.7 Ω, find the
(a) the current drawn from the battery, (b) the current in each 15.0 Ω,
(c) the terminal voltage of the battery.

Module V
11

(a) For parallel group resistance 𝑅1 we have

1 1 1 1
= + + 𝑜𝑟 𝑅1 = 5.0 Ω
𝑅1 15.0 Ω 15.0 Ω 15.0 Ω

then

𝑅𝑒𝑞 = 5.0 Ω + 0.3 Ω + 0.7 Ω = 6.0 Ω

And

Ԑ 24 𝑉
𝐼= = = 4.0 𝐴
𝑅𝑒𝑞 6.0 Ω

(b) Method 1
The three-resistor combination is equivalent to 𝑅1 = 5.0 Ω. A
current of 4.0 𝐴 flows through it. Hence the p.d. across the
combination is

𝐼𝑅1 = (4.0 𝐴)(5.0 Ω) = 20 V

This is also the p.d. across each 15 Ω resistor. Therefore, the


current through each 15 Ω resistor is

𝑉 20 𝑉
𝐼15 = = = 1.3 𝐴
𝑅 15 Ω
Method 2
In this special case, we know that one-third of the current will
go through each 15 Ω resistor. Hence

4.0 𝐴
𝐼15 = = 1.3 𝐴
3
(c)

𝑇𝑒𝑟𝑚𝑖𝑛𝑎𝑙 𝑝. 𝑑. = Ԑ − (𝐼𝑟) = 24 𝑉 − (4.0 𝐴)(0.7 Ω) = 21.2 𝑉

4. Find the equivalent resistance between points a and b for the


combination shown in Figure 4.a.

Module V
12

Figure 4.a Figure 4.b

The 3 Ω and 2 Ω resistors are in series and are equivalent to a 5 Ω


resistor. The equivalent 5 Ω is in parallel to 6 Ω , and their
equivalent 𝑅1 , is

1 1 1
= + = 0.20 + 0.167 = 0.367Ω−1 𝑜𝑟 𝑅1 = 2.73 Ω
𝑅1 5 Ω 6 Ω

The circuit thus far reduced is shown in Figure 4.b.

The 7 Ω and 2.73 Ω are equivalent to 9.73 Ω. Now the 5 Ω, 12 Ω and


9.73 Ω are in parallel, and their equivalent 𝑅2 is

1 1 1 1
= + + = 0.386Ω−1 𝑜𝑟 𝑅2 = 2.6 Ω
𝑅2 5 Ω 12 Ω 9.73 Ω

This 2.6 Ω is in series with 9 Ω, thus the equivalent resistance 𝑅𝑎𝑏

𝑅𝑎𝑏 = 9 Ω + 2.6 Ω = 11.6 Ω

5. A current of 5 𝐴 flows into the circuit of Figure 4.b at point a and out
at point b. (a) What is the potential difference from a to b? (b) How
much current flows through the 12 Ω resistors?

(a) Voltage drop from point a to b = 𝐼𝑅𝑎𝑏 = 5 𝐴(11.6 Ω ) = 58 𝑉

The voltage drop from a to c 𝑖𝑠 (5 𝐴)(9 Ω ) = 45 𝑉, hence the voltage


drop from c to b is

58 𝑉 − 45 𝑉 = 13 𝑉

(b) and the current in 12 Ω resistor is

𝑉 13 𝑉
𝐼12 = = = 1.1 𝐴
𝑅 12 Ω

Module V
13

6. In the figure below, (a) determine the equivalent resistance and (b)
the current 𝐼 through the battery.

The 5 Ω and 19 Ω are in series, their joint resistance is 24 Ω. Then 24 Ω is


parallel with 8 Ω, their joint resistance is 𝑅1 is given as

1 1 1
= + 𝑜𝑟 𝑅1 = 6 Ω
𝑅1 24 Ω 8 Ω

Now 𝑅1 = 6 Ω is in series with 15 Ω, their joint resistance is 21 Ω. Then


21 Ω is in parallel with 9 Ω, their joint resistance 𝑅2

1 1 1
= + 𝑜𝑟 𝑅2 = 6.3 Ω
𝑅2 21 Ω 9 Ω

Therefore, the equivalent resistance of the entire circuit is

(𝑎) 𝑅𝑒𝑞 = 6.3 Ω + 2 Ω + 0.2 Ω = 8.5 Ω

and the battery current


Ԑ 17 𝑉
(𝑏) 𝐼= = = 2.0 𝐴
𝑅𝑒𝑞 8.5 Ω

Module V
14

 LEARNING ACTIVITY
Name&Section:_______________________________Date:______Score:_____
Subject: ENSC 102 LA#12 Instructor: Engr. Mary Jean C. Armiendo

Direction: At the space below, show your complete solution to gain full
credits. Wrong solution means incorrect answer. Box your final answer. Use
this page as your answer sheet. You may use additional sheets if necessary.

1. For the given network, find the equivalent resistance, 𝑅𝑒𝑞 . Show
detailed solution.

2. For the circuit shown in the figure, find

a) 𝐼 in 20 Ω
b) 𝑉 at 20 Ω
c) 𝐼 in 75 Ω
d) 𝑉 at 75 Ω
e) 𝐼 in 300 Ω
f) 𝑉 at 300 Ω

3. Three resistors, of 40 Ω, 60 Ω, and 120 Ω are connected in parallel and


this parallel group is connected with 15 Ω in series with 25 Ω. The
whole system is then connected to a 120 V source. Determine (a) the 𝐼
in 25 Ω, (b) the potential drop across the parallel group, (c) the
potential across the 25 Ω, (d) the 𝐼 in the 60 Ω, and (e) the 𝐼 in 40 Ω.

Module V
15

Lesson 3

 Kirchhoff’s Laws

KIRCHHOFF’S NODE (OR JUNCTION) RULE / KIRCHHOFF’S CURRENT LAW


(KCL)
The sum of all the currents coming into a point or node (i.e. a junction
where three or more current-carrying leads attach) must equal the sum of all
currents leaving the point.

KIRCHHOFF’S LOOP (OR CIRCUIT) RULE / KIRCHHOFF’S VOLTAGE LAW


(KVL)
As one traces out a closed circuit, the algebraic sum of the potential
changes is zero. In this sum, a potential rise is positive and a potential drop
is negative.
Current always flows from high to low potential through a resistor. As
one traces through a resistor in the direction of the current, the potential
change is negative because it is a potential drop.
The positive terminal of a pure emf sources is always the high-potential
terminal, independent of the direction of the current through the emf source.

THE SET OF EQUATIONS OBTAINED by use of Kirchhoff’s loop rule will


be independent provided that each new loop equation contains a voltage
change not included in a previous equation.

Example Problems:

1. Find the currents in the circuit shown in Fig.1.

Fig.1

This circuit cannot be reduced further because it contains no


resistors in simple series or parallel. We therefore revert to Kirchhoff’s
rules. If the currents had not been labeled and shown by arrows, we
would do that first. No special care need be taken in assigning the
current directions, since those chosen incorrectly will simply give
negative numerical values.

Module V
16

Apply the point rule to point b in Fig.1:

Current into b = current out of b

𝐼1 + 𝐼2 + 𝐼3 = 0 (1)

Apply the loop rule to loop adba. In volts,

10
−7 𝐼1 + 6 + 4 = 0 𝑜𝑟 𝐼1 = 𝐴
7

(Why must the term 7 I1 have the negative sign?)

Apply the loop rule to loop abca. In volts,

12
−4 − 8 + 5 𝐼2 = 0 𝑜𝑟 𝐼2 = 𝐴
5

(Why must the signs be as written?) Now return to (1) to find

10 12 −50 − 84
𝐼3 = −𝐼1 − 𝐼2 = − − = = −3.83 𝐴
7 5 35

The negative sign tells us that 𝐼3 is opposite in direction to that shown


in the figure.

2. In Fig.2, find 𝐼1 , 𝐼2 and 𝐼3 if switch S is (a) open (b) closed.

(a) When S is open, 𝐼3 = 0, because no current can flow through the


open switch.

Apply the point rule to point a.

𝐼1 + 𝐼3 = 𝐼2 𝑜𝑟 𝐼2 = 𝐼1 + 0 = 𝐼1 (1)

Apply the loop rule to loop acbda. In volts,

−12 + 7𝐼1 + 8 𝐼2 + 9 = 0

Because 𝐼2 = 𝐼1 (1) becomes

Module V
17

15 𝐼1 = 3 𝑜𝑟 𝐼1 = 0.20 𝐴

Also, 𝐼2 = 𝐼1 = 0.20 𝐴. Notice that is the same result that one would
obtain by replacing the two batteries by a single 3 𝑉 battery.

(b) With S closed, 𝐼3 is no longer known to be zero. Applying the point


rule to point a gives

𝐼1 + 𝐼3 = 𝐼2 (2)

Appling the loop rule to loop acba gives, in volts.

−12 + 7 𝐼1 − 4 𝐼3 = 0 (3)

And to loop adba gives

−9 − 8 𝐼2 − 4 𝐼3 = 0 (4)

Note: Applying the loop rule to the remaining loop, acbda, would yield
a redundant equation, because, among the three loop equations, each
voltage change would appear twice.
We must now solve (2),(3), and (4) for 𝐼1 , 𝐼2 ,and 𝐼3 . From (4)

𝐼3 = −2 𝐼2 − 2.25

Substituting this in (3) gives

−12 + 7 𝐼1 + 8 𝐼2 + 9 = 0 𝑜𝑟 7 𝐼1 + 8 𝐼2 = 3

Also substituting for I3 in (2 given

𝐼1 − 2 𝐼2 − 2.25 = 𝐼2 𝑜𝑟 𝐼1 = 3 𝐼2 + 2.25

Substituting this value in the previous equation

21 𝐼2 + 15.75 + 8 𝐼2 = 3 𝑜𝑟 𝐼2 = −0.44 𝐴

Using this in equation for 𝐼1 gives

𝐼1 = 3(−0.44) + 2.25 = −1.32 + 2.25 = 0.93 𝐴

Notice that the minus sign is a part of the value we have found for 𝐼3 .
It must be carried along with its numerical value. Now we can use (2),
to find

𝐼3 = 𝐼2 − 𝐼1 = (−0.44) − 0.93 = −1.73 𝐴

Module V
18

3. Each of the cells shown in Fig. 3 has an emf of 1.50 𝑉 and a 0.0750 Ω
internal resistance. Find 𝐼1 , 𝐼2 , and 𝐼3 .

Applying the node rule to point a,

𝐼1 = 𝐼2 + 𝐼3 (1)

Applying the loop abcea gives, in volts,

−(0.0750) 𝐼2 + 1.5−(0.0750) 𝐼2 + 1.5 − 3𝐼1 = 0


3 𝐼1 + 0.15 𝐼2 = 3 (2)

Also, for loop adcea,

−(0.0750) 𝐼3 + 1.5−(0.0750) 𝐼3 + 1.5 − 3𝐼1 = 0


3 𝐼1 + 0.15 𝐼3 = 3 (3)

Solve eq. (2) for 3 𝐼1 and substitute in(3), yielding

3 − 0.15 𝐼2 + 0.15 𝐼3 = 3 𝑜𝑟 𝐼2 = 𝐼3

as we might have guessed from the symmetry of the problem. Then (1) yields
𝐼1 = 2 𝐼2
Substituting this in (2) gives

6 𝐼2 + 0.15 𝐼2 = 3 𝑜𝑟 𝐼2 = 0.488 𝐴
Then
𝐼3 = 𝐼2 = 0.488 𝐴

𝐼1 = 2 𝐼2 = 0.976 𝐴

4. The currents are steady in the circuit of Fig. 4. Find 𝐼1 , 𝐼2 , 𝐼3 , 𝐼4 , 𝐼5


and the charge on the capacitor.

Module V
19

The capacitor passes no current when charge and so 𝐼5 = 0. Consider


loop acba. The loop rule gives

−8 + 4 𝐼2 = 0 𝑜𝑟 𝐼2 = 2 𝐴

Using loop adeca gives

−3 𝐼1 − 9 + 8 = 0 𝑜𝑟 𝐼1 = −0.33 𝐴

Apply the node rule at point c,

𝐼1 + 𝐼5 + 𝐼2 = 𝐼3 𝑜𝑟 𝐼3 = 1.67 𝐴

And at point a:
𝐼3 = 𝐼4 + 𝐼2 𝑜𝑟 𝐼4 = −0.33 𝐴

(We should have realized that at once, because 𝐼5 = 0 and so 𝐼4 = 𝐼1 )


To find the charge on the capacitor, we need the voltage across it, 𝑉𝑓𝑔 .

Applying the loop rule to loop dfgced gives

−2 𝐼5 + 𝑉𝑓𝑔 − 7 + 9 + 3 𝐼1 = 0 𝑜𝑟 0 + + 𝑉𝑓𝑔 − 7 + 9 − 1.0 = 0

From which 𝑉𝑓𝑔 = −1 𝑉. The negative sign tells us that plate 𝑔 is


negative.

The capacitor’s charge is

𝑄 = 𝐶𝑉 = (5 𝜇𝐹)(1 𝑉) = 5 𝜇𝐶

5. For the circuit shown in Fig. 5, the resistance 𝑅 is 5 Ω and 𝜀 = 20 𝑉.


find the readings of the ammeter and the voltmeter. Assume the
meters to be ideal.

The ideal voltmeter has infinite resistance and so it can be removed


from the circuit with no effect. Let us write the loop equation for loop
cdefc:
−𝑅 𝐼1 + 12 − 8 − 7 𝐼2 = 0
which becomes

Module V
20

5 𝐼1 + 7 𝐼2 = 4 (1)

Next write the loop equation for loop cdeac. It is

−5 𝐼1 + 12 + 2 𝐼3 + 20 = 0 𝑜𝑟 5 𝐼1 − 2 𝐼3 = 32 (2)

But the node rule applied at e gives

𝐼1 + 𝐼3 = 𝐼2 (3)

Substituting (3) in (1) gives

5 𝐼1 + 7 𝐼1 + 7 𝐼3 = 4

Solve this for 𝐼3 and substitute in (2):

4 − 12 𝐼1
5 𝐼1 − 2 ( ) = 32
7

Which yields 𝐼1 = 3.9 𝐴, which is the ammeter reading.

Then (1) gives 𝐼2 = −2.2 𝐴.

To find the voltmeter reading, 𝑉𝑎𝑏 , write the loop equation for loop
abca:
𝑉𝑎𝑏 − 7 𝐼2 − 𝜀 = 0

Substituting the known values of 𝐼2 and 𝜀, then solving, we


obtain 𝑉𝑎𝑏 = 4.3 𝑉. Since this is the potential charge from a to b, point
b must be at the higher potential.

6. In the circuit of Fig. 5, 𝐼1 = 0.20 𝐴 and 𝑅 = 5 Ω. Find 𝜀.

Write the loop equation for the loop cdefc.

−𝑅 𝐼1 + 12 − 8 − 7 𝐼2 = 0 𝑜𝑟 − (5)(0.20) + 12 − 8 − 7 𝐼2 = 0

From which 𝐼2 = 0.43 𝐴. we can now find 𝐼3 by applying the node rule
at e.
𝐼1 + 𝐼3 = 𝐼2 𝑜𝑟 𝐼3 = 𝐼2 − 𝐼1 = 0.23 𝐴

Now apply the loop rule to loop cdeac.

−(5)(0.20) + 12(2)(0.23) + 𝜀 = 0

From which 𝜀 = −11.5 𝑉.

The negative sign tells us that the polarity of the battery is actually
the reverse of that shown.

Module V
21

 LEARNING ACTIVITY
Name&Section:_______________________________Date:______Score:_____
Subject: ENSC 102 LA#13 Instructor: Engr. Mary Jean C. Armiendo

Direction: At the space below, show your complete solution to gain full
credits. Wrong solution means incorrect answer. Box your final answer. Use
this page as your answer sheet. You may use additional sheets if necessary.

1. For the circuit shown in the figure below, find the currents I1, I2, and
I3. (You can simplify first the circuit before solving).

2. For the circuit shown in the figure below, find the current in the 0.96
Ω resistor and the terminal voltages of the batteries.

Module V
22

 MODULE SUMMARY

In module V, you have learned about equivalent resistance of simple


circuits and Kirchhoff’s laws. You have learned how to solve and analyze the
given circuits with worded problems.

There are three lessons in module V. Lesson 1 deals with simple circuits
with resistors connected in series and parallel.

Lesson 2 consist resistors combination. Resistors connected in series-


parallel and parallel- series.

Lesson 3 consist of Kirchhoff’s laws. Deals with equations involving


voltages and currents batteries and applying the Kirchhoff’s rules.

Congratulations! You have just studied Module V. Now you are ready to
evaluate how much you have benefited from your reading by answering the
summative test. Good Luck!!!

Module V

You might also like